Visa fråga/svar

 

Kraft-Rörelse [15733]

Fråga:
Häromdagen började jag fundera på det här med en "evighetsmaskin" Fick idén av en fysiklektion vi hade. Eftersom det inte går att rita upp så bra här ska jag försöka att förklara.

Tänk dig en pinne som sticker up en liten bit från marken som kan rotera. (pinnen sitter då fast i någon sorts bas för stabilitet)

På denna pinnen sticker det ut en magnet laddad S (d.v.s. inte laddad nord) En stavmagnet kan ju ha nord och sydpoler.

Runt den här magneten placeras 4st magneter laddade N. Varje magnet är placerade med 90 graders vinkel från varandra. D.v.s. 4st N magneter runt S magneten.

Som ni vet attraheras Nord av Syd och vice versa. Så idén är att ha hyfsat starka magneter så att om jag skulle ha 1st Nord magnet och jag har S magneten ett kvarts varv bort så ska den attraheras till N magneten.

Men hur är detta en evighetsmaskin? Jo ja hade tänkt att på den här roterande pinnen med magneten i mitten också fästa ett ämne som neutraliserar magnetfält.

Detta ämne sitter fast på samma pinne som magneten och roterar då med den.

Detta ämne skulle vara cirkel format och ha tillräcklig stor bredd för att påverkan av N magneten utanför ämnet inte ska påverka S magneten så att den fortsätter mot nästa magnet.

Har här försökt att illustrera hur jag tänkt. Den ska självklart vara helt cirkelformad och inte oval som bilden. Det som jag försöker visa är att detta ämne som blockerar magnetfältet mellan S och N (se höger sida av bilden) finns en bit framför S.Det är en öppning för att när pinnen snurrar så vill jag att den ska attraheras av nästa magnet och sen nästa D.v.s. hålla uppe farten. Om jag inte hade detta ämne så skulle den stanna vid första magnet.

N ----- / \ ! / \ ! N ! O------S ! N ! / / \ / / !---------------/ N

Så min fråga är. Varför funkar inte detta?

Eftersom jag vet att det strider mot energilagarna. Frågade min fysiklärare och han trodde att det kunde ha något med virvel strömmar mellan det blockerande ämnet och N magneterna.

Hoppas ni förstod hur jag menade.
/Robert L, porthälla

Svar:
En evighetsmaskin av första ordningen bryter mot termodynamikens första huvudsats:

Lagen om energins bevarande: energi i ett slutet system inte kan skapas eller förintas.

En evighetsmaskin av andra ordningen bryter mot termodynamikens andra huvudsats som kan formuleras på olika sätt:

Alla processer som kan förekomma i ett isolerat system leder till att entropin ökar eller möjligtvis förblir konstant.

Ingen process är möjlig vars enda resultat är att värme tas från en reservoar och helt omvandlas till arbete. (Lord Kelvin)

Ingen process är möjlig vars enda resultat är att värme överförs från en kallare till en varmare kropp. (Clausius)

Första och andra huvudsatsen har uttryckts elegant och generellt: Nothing disappears, and Everything spreads (Ingenting försvinner, och allting sprids).

Entropi (makroskopisk definition) är en till värme kopplad termodynamisk tillståndsstorhet och är ett mått på hur mycket av värmeenergin som i en värmemotor ovillkorligen måste avges vid nedre temperaturen och således aldrig kan omvandlas till arbete, se Entropi . Entropiändringen dS ges av överförda värmen dQ och absoluta temperaturen T:

dS = dQ/T

Se vidare Laws_of_thermodynamics och Termodynamik .

Jag förstår inte exakt hur din maskin är uppbyggd, men den måste vara en första ordningens evighetsmaskin. Detta eftersom det måste finnas friktion i den roterande pinnen.

Eftersom energins bevarande är så central inom fysiken har man vad gäller patent (åtminstone i USA) på evighetsmaskiner ett starkare krav än normalt: man måste kunna visa upp en fungerande prototyp, se Perpetual_motion#Patents . Hittills har ingen lyckats.

Observera att termodynamikens lagar är, liksom alla fysikaliska lagar, ingenting man kan härleda från "axiom" som i matemaiken. Alla fysikaliska lagar baseras på experiment och observationer. Energiprincipen (första huvudsatsen) uppfylls i alla experiment man utfört, och får anses mycket etablerad. Man har faktiskt vid några tillfällen föreslagit att man måste överge energiprincipen, t.ex. när man observerade ett kontinuerligt spektrum av elektroner vid beta-sönderfall. I detta fallet löstes problemet när man upptäckte att ytterligare en partikel var involverad vid beta-sönderfallet: neutrinen.

Jag kan som sagt inte exakt säga vad som är svagheten i din konstruktion, men jag har ett par kommentarer:

  • Som din lärare säger, det kommer att induceras virvelströmmar i din magnetfältssköld. Förutom att även de ger energiförluster (uppvärmning) så inducerar de även magnetfält som motverkar rörelsen. Du kan alltså med t.ex. mymetall bli av med magnetfältet bakom mymetallen, men du har alltid en växelverkan mellan mymetallen och magneten.
  • Du säger att du använder magneter laddade N. Vi har aldrig observerat monopoler, så det är tveksamt att de existerar. Detta är emellertid egentligen inget skäl till att din konstruktion inte skulle fungera - nuvarande teorier tillåter monopoler, så de kan knappast orsaka brott mot energiprincipen.
Se vidare Perpetual_motion varifrån nedanstående exempel på konstruktioner av evighetsmaskiner kommer. Se även Donald Simaneks "The Museum of Unworkable Devices" under länk 1.


/Peter E

Nyckelord: evighetsmaskin [14]; termodynamik [17]; entropi [7];

1 http://www.lhup.edu/~dsimanek/museum/unwork.htm

*

 

 

Frågelådan innehåller 7624 frågor med svar.
Senaste ändringen i databasen gjordes 2022-05-21 17:33:39.

 

** Frågelådan är stängd för nya frågor tills vidare **


sök | söktips | Veckans fråga | alla 'Veckans fråga' | ämnen | dokumentation | ställ en fråga
till diskussionsfora

 

Creative Commons License

Denna sida från NRCF är licensierad under Creative Commons:
Erkännande-Ickekommersiell-Inga bearbetningar
.